4
$\begingroup$

Let $f:\mathbb{R} \to \mathbb{R}$ be a smooth function with bounded derivative. Define the Nemytskii map $F:H^1(\Omega) \to H^1(\Omega)$ by $F(u)(x) := f(u(x))$. Here $\Omega$ is a bounded smooth domain.

There exists work where we can deduce continuity and differentiability of $F$ under some assumptions on $f$.

What conditions on $f$ will ensure that $F$ satisfies the following compactness criterion: if $u_n \rightharpoonup u$ in $H^1(\Omega)$ (weakly), then there is a subsequence of the $u_n$ such that $F(u_{n_j}) \to F(u)$ strongly in $H^1(\Omega)$?

If convergence is $H^1$ is too difficult, $L^2$ (in both domain and range of $F$) can be ok. Does anyone come across this before?

$\endgroup$

2 Answers 2

4
$\begingroup$

I think in your setting $F$ is not compact, unless $f$ is a constant map. Indeed, if $f$ is not a constant map, it coincides on some non-trivial interval $[a,b]$ with a smooth diffeomorphism $g:\mathbb{R}\to\mathbb{R}$ with derivatives $g'$ and $(g^{-1})'$ bounded on $\mathbb{R}$. Thus the corresponding Nemytskii map $G:u\mapsto g\circ u$ is a homeomorphism of $H^1(\Omega)$ into itself.

The set $E$ of all $u$ in the unit ball of $H^1(\Omega)$ with $a\le u \le b$ is of course bounded, but never relatively compact in $H^1$, so neither is $G(E)$. But the latter set coincides with $F(E)$, so $F$ is not a compact map in the said sense.

$\endgroup$
1
  • $\begingroup$ For the non-compactness of the set $E$: start with a sequence $u_j\in H^1$ with $\|\nabla u_j\|_2=1$ and $u_j \to 0$ a.e. If $f(t):=\mathrm{dist}(t,\mathbb{Z})$, then $v_j:=f(u_j)$ has still $\|\nabla v_j\|_2=1$, $v_j \to 0$ a.e. and also $\|v_j\|_{\infty}\le1$. $\endgroup$ May 3, 2017 at 11:33
4
$\begingroup$

There is a general theorem about superposition (i.e., Nemytskii) operators between spaces of measurable functions: If the measure is non-atomic, the superposition operator defined by $f$ is compact if and only if $f$ is constant, see Theorem 1.8 in

Appell, Jürgen; Zabrejko, Petr P., Nonlinear superposition operators, Cambridge Tracts in Mathematics 95. Cambridge: Cambridge University Press (ISBN 978-0-521-09093-3/pbk). 311~p. (2008). ZBL1156.47052.

$\endgroup$
1
  • $\begingroup$ (some argument is still needed to pass from the setting of measurable functions to $H^1$) $\endgroup$ May 3, 2017 at 11:36

Your Answer

By clicking “Post Your Answer”, you agree to our terms of service and acknowledge you have read our privacy policy.

Not the answer you're looking for? Browse other questions tagged or ask your own question.